8
$\begingroup$

For any collection of permutations of $\{1,2,\dots,n\}$, we say that it realizes a directed multigraph with $1,2,\dots,n$ as vertices, such that there is an edge from $i$ to $j$ if $i$ appears before $j$ in at least half of the permutations, and there is an extra edge from $i$ to $j$ (so, two edges in total) if $i$ appears before $j$ in all of the permutations.

Is it true that for any collection of permutations that realizes a multigraph $G$, there exists a collection of two permutations (not necessarily from the original collection) that realizes a multigraph $H$ such that the edge set of $H$ is a superset of the edge set of $G$?

For example, if we have a collection of permutations

$(1,2,3,4), (1,4,3,2), (2,3,1,4), (3,1,2,4)$

the graph $G$ contains two edges $(1,4)$ and one edge $(1,2), (1,3), (2,3), (2,4), (3,1), (3,2), (3,4)$ each. Then we can choose the collection

$(1,2,3,4), (3,1,2,4)$

to fulfill the condition, since the realized graph $H$ contains two edges $(1,2), (1,4), (2,4), (3,4)$ each and one edge $(1,3), (2,3), (3,1), (3,2)$ each.

$\endgroup$
5
  • $\begingroup$ I tried for 10 mins to build a counter example unsuccesfully. Here are my observations though: when you take any 2 perms and build $H$ the graph is complete with double edges. The only bad thing that can happen is that a double edge is in the wrong direction. So a CE has for any pair of permutation a pair of vertex that is (put simply) in the other direction for the other permutation. I think a simple CE should have 5 permutations in which you have a lot of single edges s.t. they exist in a 2-3 distribution. Propably (not sure) you need at least 6 vertices $\endgroup$ Commented Jul 1, 2024 at 16:09
  • 1
    $\begingroup$ The permutation (1 2) can also be written as (2 1). Your multigraph is not well-defined. There are a few notations noted in the [permutation][1] Wikipedia article. Do you want e.g. to use the one-line notation or the canonical cycle notation? [1]: en.wikipedia.org/wiki/Permutation#Notations $\endgroup$ Commented Jul 4, 2024 at 2:17
  • 1
    $\begingroup$ @TheAlertGerbil I'm using the one-line notation. $\endgroup$ Commented Jul 4, 2024 at 11:05
  • $\begingroup$ Can our initial collection have size 1? (1,2,3) realises a graph with 2 edges (1,2), (2,3). If you have 2 permutations, one of those permutations must have either a 2 before a 1 or a 3 before a 2, so it cannot realise a larger edge set. This would be a counterexample. $\endgroup$ Commented Jul 26, 2024 at 14:53
  • 1
    $\begingroup$ @chessman If the initial collection is just $(1,2,3)$, then $G$ would have two $(1,2)$, two $(1,3)$, and two $(2,3)$. We can choose two permutations $(1,2,3)$, which make $H$ the same as $G$. (The two permutations can be identical.) $\endgroup$ Commented Jul 26, 2024 at 15:45

2 Answers 2

1
$\begingroup$

I don't have a full proof, but I wand to share some thoughts and ideas to start a discussion.

Firstly note that each directed acyclic graph corresponds to a permutation, which can be found as a topological ordering of this graph.

Also set of edges of a simple digraph can be divided into two subsets inducing an acyclic graph each. For this purpose choose any vertex ordering along a line, then collect arcs going from left to right into one subset, and arcs going from right to left into another one (let's call it left-right separation). Therefore as long as digraph $G$ is simple the permutations $(1, 2, \ldots, n)$ and $(n, n - 1, \ldots, 1)$ induce its supergraph $H$.

So a potential problem comes with double arcs only. Here we need to divide set of single arcs into two subsets such that each of two subsets being united with the set of double arcs induce an acyclic graph. Here we can't take an arbitrary ordering of vertices to use left-right separation as before.

An example is the following: permutations $(1, 2, 3)$ and $(3, 1, 2)$ induce a graph $G$ with arc multiset $\{\,(1, 2), (1, 2), (1, 3), (2, 3), (3, 1), (3, 2)\,\}$. And a bad ordering for left-right arc separation is $1, 3, 2$: two subsets are $\{\,(1, 3), (3, 2)\,\}$ and $\{\,(2, 3), (3, 1)\,\}$. The first subset perfectly unites with the arc $(1, 2)$. But the second one produces a cycle $1, 2, 3, 1$. However two other vertex orderings, $1, 2, 3$ and $2, 1, 3$, are good for left-right separation, and remaining three are essentially the same as these three.

This example shows that it makes sense to require that vertices of the same weak component of graph $G_{\mathrm D}$, induced by double arcs of $G$, form a contiguous subsequence of vertex ordering. But it is not enough: consider permutations $(1, 2, 3, 4)$ and $(1, 4, 2, 3)$. This example is essentially the same as the previous one, but all vertices now belong to the same weak component in $G_{\mathrm D}$.

Here I give another approach. It could be wrong, feel free to challenge it. Let's construct a vertex ordering in the following way. For every position from $1$ to $n$ we select a vertex with no incoming double arc going from a remaining vertex. Among all such vertices we give priority to a vertex with maximum number of incoming double arcs from previously selected vertices. After getting such an ordering let's do left-right separation of single arcs. One subset being united with double arcs set definitely induces an acyclic graph, since all double arcs go from left to right. And I suppose that another one being united with double arcs set also induces an acyclic graph, but a proof is not ready at this moment.

An important property of the graph $G_{\mathrm D}$ is that it is a transitive closure of an acyclic digraph. The property about transitive closure comes from an observation that if $x$ comes before $y$ in all permutations and $y$ comes always before $z$ than $x$ always comes before $z$. And existence of a cycle leads to a contradiction of form "$x$ always comes before itself".

$\endgroup$
0
$\begingroup$

I believe the result is true. Consider first what sort of graphs are realised by collections of 2 permutations. For a permutation $\sigma$, let $E_\sigma = \{(\sigma(i),\sigma(j)): 1 \leq i < j \leq n \}$ be the "corresponding edge set". Then one way of interpreting the multigraph we form from a collection of 2 permutations $\sigma$ and $\sigma'$ is that it is the graph with edge set $E_\sigma \sqcup E_\sigma'$. Consequentially, for any pair of vertices, either $x \Rightarrow y$, $y \Rightarrow x$ or $x \leftrightarrow y$ (I hope the meaning behind this notation is clear here!)

Thus, our strategy is simple. For any collection of permutations which realises a graph G, it is sufficient to pick our 2 permutations to realise $H$ such that whenever $x \Rightarrow y$ in $G$, $x \Rightarrow y$ in $H$, and otherwise, $x \leftrightarrow y$. Then $H$'s edge set covers that of $G$.

To construct the two permutations, label the vertices $\{v^i_{j}: 1 \leq j \leq n_i\}$ (where $i$ ranges from $1$ to some $k$ and $n_1 + \dots + n_k = n$) such that if we look at the subgraph of G comprising of edges which appear twice (which we note is a forest), the sets $\{v^i_{j}: 1 \leq j \leq n_i\}$ are the trees (possibly singletons) that make up the forest. Then, with a bit of thought, we can realise that the required permutations can be chosen to be $\sigma = (v^1_1,\dots,v^1_{n_1},\dots, v^k_{1},\dots, v^k_{n_k})$ and $\sigma' =(v^k_1,\dots,v^k_{n_k},\dots, v^1_{1},\dots, v^1_{n_1})$ (we reverse the order of the trees). It is not hard to see that vertices within the same tree appear as double edges in the realised graph, while vertices not in the same tree have edges both ways in the realised graph.

Apologies if I've misunderstood something! Just let me know.

$\endgroup$
1
  • $\begingroup$ The order inside weak components induced by double arcs (which are not trees, but transitive closures of acyclic directed graphs) is also important. You can find an example of a bad order in my post $\endgroup$ Commented Jul 29, 2024 at 16:10

You must log in to answer this question.

Start asking to get answers

Find the answer to your question by asking.

Ask question

Explore related questions

See similar questions with these tags.